Giáo trình Sáng tạo bất đẳng thức

Lời nói đầu V

Cộng tác viên xi

1 Bất đẳng thức Cơ Sỡ 3

1.1 Bất đẳng thức AM-GM 4

1.1.1 Bất đẳng thức AM-GM và ứng dụng 4

1.1.2 Kĩ thuật Côsi ngược dấu 13

1.2 Bất đẳng thức Cauchy-Schwarz-Holder 18

1.2.1 Bất. đẳng thức Cauchy-Schwarz và ứng dụng 18

1.2.2 Bất đẳng thức Holder 27

1.3 Bất đẳng thức Chebyshev 34

1.3.1 Bất đẳng thức Chebyshev và ứng dụng 34

1.3.2 Kĩ thuật phân tách Chebyshev 39

1.4 Bốt đẳng thức với hàm lồi . 44

1.4.1 Hàm lồi với bất đẳng thức Jensen 44

1.4.2 Hàm lồi với kĩ thuật xét phần tử ở biên 49

1.5 Khai triển Abel và bất đẳng thức hoán vị 53

1.5.1 Khai triển Abel 53

1.5.2 Bát đẳng thức hoán vị * . . . . 59

1.6 Bất đẳng thức đối xứng 3 biến 61

1.6.1 Bất đẳng thức thuần nhất không có điều kiện _ 62

1.6.2 Bất đẳng thức đối xứng có điều kiện 66

 

docx362 trang | Chia sẻ: trungkhoi17 | Lượt xem: 404 | Lượt tải: 1download
Bạn đang xem trước 20 trang tài liệu Giáo trình Sáng tạo bất đẳng thức, để xem tài liệu hoàn chỉnh bạn click vào nút DOWNLOAD ở trên
Nhận xét. Cách chứng minh trên được suy ra từ nhận xét đơn giản sau : Nếu thay x,y, z bởi X - t,y - t, z - t thì VP - VT của (1) giảm dần, do đó khi ta lấy t đủ lớn để x,y, z suy biến thành độ dài 3 cạnh của một tam giác vuông thì bất đẳng thức ban đầu trở về trường hợp đơn giản là có một trong 3 số a, b, c bằng 0. Bài toán 2.90 (Phạm Kim Hùng). Với a,b,c là các số thực tuỳ ý, chứng minh bất đẳng thức sau đây 1 1 1 11 (2ữ - ờ)2 + (2Ò - c)2 + (2c - a)2 ~ 7(ữ2 + b2 + c2) LỜI Giải. Đặt X — 2a - b,y = 2b - c, z = 2c - a. khi đó 4x + 2y + z L 4y + 2z + x 4z + 2x + y a — —-7 , b = — , c = - 7 7 Suy ra X2 + y2 + z = — 7 . Ta phải chứng minh 1 J_ _Ị_ 11 x2\2 + z2 ~ 2[x + y + z)2 + X2 + y2 + z2 Do tính đối xứng, có thể giả sử ràng X, y •> 0 > z. Nói cách khác ta phải chứng minh bất đẳng thức (sau khi đã đổi dấu của z) (^> y, Z) x2 y2 Z1 2(x + y - z)2 + X2 + y2 + z2 ~ (ỉ). Nếu z > X + y, ta chứng minh f(x, y, z) > f(x, y,x + y). Thật vậy f(x,y,z) - f(x,y, X + y) = ll(3z - X - y) z - X — y (x2 + y2 + (x + y)2)(x2 + y2 + z2 + 2(z — X — y)2) x + y + z z2(x + y)2 " (Sử dụng các bất đẳng thức sau 3z - X — y > X + y + z, 2(x + y)2 > X2 + y2 + (x + y)2, ■ 4z2 > X2 + y2 + z2 + 2(z — X —■ y)2). Ngoài ra dễ thấy răng /2.2./-, ,a2x /1.1 1 A 27 (a; + ỳ + (^ + ỹ) ) I ~2 + ~2 + 7Z-7772 ) — o’ \x2 y2 (x + y)2/ 2 Do đó f(x, y, X + y) > 0. Đó là điều phải chứng minh. (ịị). Nếu X + y > z. Khi đó f(x, y, z) > f(t, t, z) với t — + y. 1 11 *’ “ í2 + z2 2t2 + z2 + (2í - z)2 ■ Lấy z — 1, quy đồng mẫu số rồi đứa về bất đẳng thức với 1 biến của t 5í4 - 4t3 + 6t2 - 8t + 3 > 0 , t>|. Kiểm tra trực tiếp suy ra bất đẳng thức trên đúng. □ Nhận xét. Hằng số tốt nhất trong bất đẳng thức (Va, 6, c G 7?) 1 1 1 k (2a - b)2 + (26 - c)2 + (2c - a)2 - 7(a2 + b2 + c2) Là min fix) — 10x2 + X - — - 8rr + 23 với X > -. Có thể tính gần dũng giá trị này min f(x) = 11.6075. Hệ quả. Giả sứ a, b, c là các số thực tuỳ ý và (a + ò + c)(2a - b)(2b - c)(2c — a) > 0. Khi đó ta có bất đẳng thức 1 1 1 . 27 (2a - b)2 + (26 - c)2 + (2c - a)2 - 22(a2 + b2 + c2) ■ Đẳng thức xảy ra khi và chỉ khi (a, b, c) = (4Ả;, k, —5k) hoặc các hoán vị (Vk ẽ R). Thực chất đây chính là trường hợp z > X + y trong chứng minh trên. Bài toán 2.91 (Phạm Kim Hùng). 1. Cấc số thực dương a,b,c thoả mãn a3 + b3 + c3 = 3. Xét bất đẳng thức ab bc ca „ — + — + ~r > 3. cab . Chứng minh bất đẳng thức không thể luôn đúng với mọi a,b,c. (íí). Chứng minh Tằng bất đẳng thức sê đúng nếu a, b, c thoả mãn điều kiện a3b3 + b3c3 + c3a3 > abcfa3 + b3 + c3). 2. Tìm hằng số dương k lớn nhất sao cho với mọi số thực dương a, b, c bất đang thức sau luôn đứng ab bc ca „ k/ak + bk + ck „ + „ + h - Sy Q ■ c a b V 3 LỜI Giải. l.(i). Cho a = b = 0.8, c = a/E976. (ií). Theo bất đẳng thức AM - GM ta có a2b + b2a + 1 > 3ab, b2c + c2b + 1 > 36c, c2a + a2c + 1 > 3ca, => abc (ab(a + 6) + bc(b + c) + ca(c + a)) > abcíab + bc + ca). Từ đó két hợp với điều kiện a3b3 + b3c3 + c3a3 > abc(a3 + b3 + c3) = 3abc ta suy ra (ab + be + ca)(a2b2 + b2c2 + c2a2) > 3abc(ab + bc + ca) 9.9.199. 9 9 _ 1 ữb bc ca => a2b2 4- b2c2 + c2a2 > 3abc => — + — + ,>3. cab 2. Ta xét bài toán ngược lại, giả sử rằng a, b, c là các số thực thoả mãn ab bc ca . + . + A = 3’ c a b Ta cần tìm giá trị lớn nhất của s = ak + bk + ck. Đặt X — ab/c, y — bc/a, z — ca/b. Khi ãó X + y + z = 3. S^(xy)k/2A-(yz)k^2 + (zx)k/2. Theo một kết quả quen biết thì ( 3fc\ s < max I 3, XT I . \ 2k J Và do đó giá trị tốt nhất để bất đẳng thức còn đúng là = 3fc/2fc k = , oln3 „ % 2.709511... < 3. □ In 3 — In 2 Bài toán 2.92 (Vasile Cirtoaje). Các a,b,c,d là các số thực không âm thoả mãn a + b + c + d = 4. Hãy chứng minh bất đẳng thức sau 1 1 1 1 < 1 5 — abc 5 — bed 5 — cda 5 — abc ~ LỜI Giải. Đặt X = abc, y = bed, z = cda, t — dab. Ta phải chứng minh ĩ> - X 5 - y 5-2 5 - t 1 - X , 1 - y . 1-2 . 1 - t n (1 -x)(x + 2) (!-?/)(?/+2) (1 - z)(z + 2) (1 - t)(t + 2) (5 - z)(rr + 2) (5-y)(y + 2) (5-z)(2 + 2) (5 - t)(t + 2) De thấy nếu X > y thì 2 - X - X2 (1 - .r)(2 + x) < (1 - y)(2 + ỳ). Hơn nữa theo bất đẳng thức AM — GM thì X + y — beta + d) < |(ò + c + a + d)3 — < 3. ó Z í nên ta có (5 - x)(x + 2) - (5 - y)(y + 2) = (x - y)(5 - X — y) > 0. Áp dụng bất đẳng thức Chebyshev suy ra (J > ( £ (1 - + 2)) ( £ . Và bây giờ ta cần chứng minh (I - x)(x + 2) ='8 - X - y - z - t - X2 - y2 - z2 - t2 > ũ z,y,z,t fla, b, c, d) = abc + bed. + cda + dab + a2b2c2 + b2c2d2 + c2d2a2 + d2a2b2. Không mất tính tổng quát ta giả sử a > b > c > d. Đặt m = (a + c)/2, u= (a — c)/2, t' = m2, V = ú2 và g(u2) = (m2 - u2)(b + d) + 2bdm+ (m2 - u2)2(b2 + d2) + 2b2d2(m2 + u2), ,ợ(v) = (t' - v)(b + d) + (t' - v)2(b2 + d2) + 2b2d2(t' + v) + 2Vĩbd, g'(v) = —(6 + d) — 2{t' — v)(b2 + d2) + 2b2d2. Bởi vì t' — V = ac > bd nên g'(y) < 0 =» y(a, b, c, d) = g(v) < ,ợ(0). Vậy ta chứng minh bài toán khi a = b = c = a,d — 4 — 3a. Ta phải chứng minh ft3 + 3o2(4 - 3a) + a1 \/4t2 + tc t + 3o 2t + c (4 - 3a)2 (o - l)2(7a4 - 4o3 - 3o2 - 4a - 2) < 0. Bất đẳng thức trên đúng, vì a < 4/3 nên 7o4 — 4o3 — 3o2 — 4ư — 2 < 0. Bài toán đã được chứng minh xong. Đẳng thức xảy ra chỉ khi a = b = c = d=l. □ Nhận xét. Từ bất đẳng thức trên ta có thể suy ra kết quả tổng quát là Bài toán. Chứng minh với mọi dãy số thực không âm ai,a2, ...,an có tổng bằng n,n > 4 thì 1 1 1 < 1 n + 1 - &1 n + 1 — ỏ2 n + 1 - bn Trong đó bk — aia2...ak_iafc+i...an VA; = 1, n. Chứng minh bất đẳng thức bằng phương pháp quy nạp, lưu ý rằng trong lời giải đó thì xét với n = 4 lại là phần khó nhất, nhưng đã được chứng minh ở trên. Bài toán 2.93 (Phạm Kim Hùng). Chứng minh với mọi số thực a, b, c không âm ta luôn có bất đắng thức 1 1 1 4 y/ 4a2 + bc \/ 4b2 + ca V 4c2 + ab a + b + c LỜI Giải. Giả sử a > b > c. Đặt t — > c, ta có (4a2 + bc)(4ò2 + ca) < (4t2 + tc)2 o 16a2b2 + 4(a3 + b3)c + abc2 < (a + b)4 + (a + b)3c + ^(a + b)2c2 & (a - ỏ)2 I ịc2 + a2 + b2 + Qab - 3ca - 3cb ) > 0. \4 J Nhưng bất đảng thức trên hiễn nhiên đúng, vì a > b > c. Vậy = /ơ,c). 1 1 1 2 1 Ta phải chứng minh y/4a2 + be V4b2 + ca ự4c2 + ab ~ ỵ/4t2 + tc y/4c2 + t2 —c —4c2 —2c ự4t2 + íc(2í 4- ự2í2 4- tc) tự4c2 + í2(i + ự4c2 + í2) ~ t(2t + c) 2 1 4c > t(2t 4- c) \/4í2 4- tc(2t + ự2t2 4- tc) ty/4c2 4- i2(í 4- ự4c2 4- í2) - Cuối cùng ta chỉ cần chứng minh hai bất đẳng thức sau . Chứng minh 1 1 3t(2t + c) ~ y/4t2 4- tc(2t 4- ự2t2 4- tc) 9t2(2í 4- c)2 < (4t2 4- tc)(2t + y/4t2 4- tc)2 t2 4- 6tc 4- 2c2 < 2tỵ/4t2 + tc + cy/4t2 + tc. Bất đẳng thức trên hiển nhiên đúng vì t > c. . Chứng minh 4c 3t(2t 4- c) ty/4c2 4- t2(t 4- ự4c2 4-12) 12c(2t 4- c). Chú ý rằng ỵ/5(4c2 4- t2) > 4c 4-1 nên ta có VT > (4c 4- t)(4c 4- (x/5 4- l)t) > 12c(2t 4- c) 44 (ựõ 4- l)t2 4- (16 - 4\/5)tc 4- 4c2 > 0. Bất đẳng thức trên đúng, vì A < 0. Đẳng thức xảy ra khi và chỉ khi a = b, c — 0 hoặc hoán vị. □ Nhận xét. Xét bài toán mở rộng Bài toán. Tìm hằng số dương k tốt nhất dể bất đang thức sau luôn đúng với các bộ số thực không âm a, b, c tùy ý 1 1 1 4 4- 2fc \/k2a2 4- bc y/k2b2 4- ca ỵ/k2c2 4- ab ~ k(a 4- b 4- c) Bằng cách tương tự, ta tìm được k là nghiệm dương của phương trình đại số _ 4 + 2fc ự/c2 4- 1 3A: Bài toán 2.94. Chứng minh với mọi a, b,'c không âm thì CI3 Ễ>3 c3 a 4- b 4- c 2a2 4- b2 + 2b2 4- c2 + 2c2 4- a2 - 3 ■ LỜI Giải. Ta có 3a3 a(a + b)(a — ò) 2ữ2 + b2 2a2 + b2 3ữ3 2(a — 6) . ,.fa(a + b) ^2^b2~a~ -3={a-b) (m 2\ 3/ (a - ft)2 2b — a 3(2a2 + ft2) Vậy bất đẳng thức đã cho có thể viết dưới dạng 2b - a 2a2 + b2 (a - b)2 + 2c - b 2b2 + c2 (b - c)2 + 2a - c 2c2 + á2 (c — a)2 > 0. Xét các trường hợp sau đây. +, Trường hợp 1 . a > b > c. Khi đó 4ft c > n ~%a 4- > 0 2a2 + b2 2c2 + á2 ~ ’ 2a2 + b2 + 2c2 + á2 ~ 4b - 2a 2a~- c 2ơ,2 + b2 2c2 + a2 ~ 4b-2a 2a2 + b2 (a - ft)2 + 2a — c 2c2 + a2 (c - a)2 > 0 (1) (4c - 2ft)62 (2a - c)a2 2b2 + c2 + 2c2 + a2 ~ 4c - 2b 2b2 + c2 (ò - c)2 + 2a — c 2c2 + á2 (c - a)2 > 0 (2) Cộng vế (1) và (2) ta có điều phải chứng minh. +, Trường hợp 2 . c > b > a. Xét thêm 2 trường hợp nhỏ (j). Nếu 2b > a + c. Ta sẽ chứng minh 2 bất đẳng thức sau 2b - a , 2a - c „ —1—__ -L 4—Z—> Q 2a2 + b2 2c2 + a2 Thật vậy, dễ thấy vé trái là hàm tăng của c nSn chỉ cần chứng minh khi c — b, hay 2b - a 8a- 4b 2a2 + b2 + 2b2 + a2 ~ Bất đẵng thức trên tương đương với 4Ỏ3 + 2ba2 ~ 2ab2 - a3 + 16ữ3 - 8a2b + 8ab2 - 4b3 > 0 o 15ữ3 - Qa2b + 6ab2 > 0 5a2 - 2ab + 2b2 > 0 (đúng!). Do đó 2b — a 2a2 + b2 (a — b)2 + 2c - b 2b2 + c2 (Ò-C)2 + 2a — c 2c2 4- a2 (c - a)2 2b — a 4(2a2 + b2} (c - à)2 + 2a — c 2c2 + a (w). Nếu 26 < a + c, ta sẽ chứng minh 2b - a 6« - 3c . 2a2 + b2 + 2c2 + a2 - w Thật vậy, vì VT là hàm tâng của c nên chỉ cần chứng minh khi c = 2b~ a. Bất đẳng thức (3) trở thành 26 - a 9a — 66 2a2 + 62 + 862 + 3a2 - Sab ~ 1063 - 1562a + 2a?b + 15a3 > 0. Nhưng bất đẳng thức trên hiển nhiên đúng theo bất đẳng thức AM - GM 1063 + 15a3 > 3v/563.563.15a3 > 1562a 2c - 6 3 2a - c 2b2 + c2 + 2 2c2 + a2 - {) Thật vậy, vì biểu thức vế trái là hàm giảm của a nên ta chỉ cần chứng minh khi a — b, bất đẳng thức trở thành 4c - 26 66 - 3c 262 + c2 + 2c2 + 62 - 5c3 + 2c26 - 262c + 1063 > 0 (đúng!). Từ 2 bất đẳng thức trên, với chú ý ràng (c - a)2 < 3(6 - a)2 + |(c - 6)2 ta có 26 - a 2a2 + 62 (a - 6)2 + 2c - 6 262 + c2 (6 - c)2 + 2a - c 2c2 + a2 (c-a)2 26 - a o 2a - c \ 2 . f 2c - 6 3 2a - c \ 2a2 + 62 + 32c2 + a2 J ~ Ò) + \262 + c2 + 2'2c2 + a2 J (6 - c)2 > 0. Bất đẳng thức được chứng minh. Dẳng thức xảy ra khi và chỉ khi a = b = c. □ Bài toán 2.95 (IMO Shortlist 2003). Cho n là một số nguyên dương và hai dãy số thực dương (xi,«21 3?n); (yi>ỉ/2, 2/n)- Giở sử rằng (Z2, Z3t 22n) là một dãy số dương thoả mãn điều kiện Dặt M = max{z2, ..., z2n}- Chứng minh rằng / M + Z2 + ... + z2n ^^Ấ^l+^ + .-.+^nA í X1 + x2 + • + ZnA \ 2zz / \ n J X n / LỜI Giải. Đặt X = max{xi, X2, ..., xn} và Y = max{j/!,t/2> Không mất tính tổng quát, ta có thể giả sử X = Y = 1 (nếu không thay thế Xị bởi Xilx,yị bởi yiỊY, Zị bởi Zi/y/XY). Bằng bất đẳng thức Cauchy — Schwarz, ta sẽ chứng minh bất đẳng thức mạnh hơn M + Z2 + • + Z2n > Xi + X2 + ... Y xn + yi + y2 + ... + yn (*) Để chứng minh bất đẳng thức trên, trước hết ta sẽ chứng minh rằng với mọi r > 0, số các số hạng lớn hơn r trong vế phải không vượt quá số các số hạng lớn hơn r trong vế trái. Thật vậy, rõ ràng khẳng định đúng ncu r > 1 (vì không có số hạng nào ở vế phải lớn hơn r). Ta xét với r < 1. Đặt A — {i ẽ N, 1 r}, a — |4|, B = {i G N, 1 r}, b = |B|. Ta có a,b> 1. Giả sử A — {íi, Ì2,... ,ia} và B — {ji, j2> ■■■, jb} với Z1 < Z2 < ... < ia, jì < 32 < ••• < jb- Khi đó CÓ ít nhất a + b - 1 số hạng của dãy Z2, z3,Z2n lớn hơn r là zil+jl > 2Ú+j2’ 2Ú+jl>’ ZÌ2+jb’ zia+jb' Mặt khắc, vì a + b - 1 > 1 nên ít nhất có một số Zị nào đó lớn hơn r, vậy M > r. Do đó có ít nhất a + b số hạng ở vé trái của (*) lớn hơn r (đpcm). Từ tính chất trên suy ra, với mọi fc, 1 < k < 2n, số lớn thứ k trong các số hạng ở vế trái không nhỏ hơn số lớn thứ k trong các số hạng ở vế phải. Do đó đương nhiên tổng các số hạng Ư vế trái lún hơn tổng các số hạng ử vế phải. Bất đăng thức được chứng minh. □ Bài toán 2.96 (Vasile Cirtoaje, Phạm Kim Hùng), (i). Chứng minh rằng với a, b, c là các số thực tuỳ ý ta luôn có a4 + ỏ4 + c4 + ab3 + bc3 + Cữ3 > 2(ữ36 + b3c-+ c3a). . Giả sử a, b, c là các số thực dương thoả mãn a2 + ò2 + c2 + ab + bc + ca — 6. Chứng minh rằng a3b + b3c + c3a + abc(a + b + c) < 6. . Tìm hằng số k tốt nhất (lớn nhất) sao cho bất đẳng thức sau a4 + b4 + c4 + k(ab + bc + Cữ)2 > (1 + 3fc)(a3ò + b3c + c3a) luôn đúng với mọi a,b,cE R. LỜI Giải. Xét khai triển sau (x2 — kxy + kxz — z2)2 + (y2 — kyz + kyx — X2)2 + (z2 — kzx + kzy — y2)2 > 0. Từ khai triển trên ta suy ra X4 +y4 + z4 + (k2 — l)(x2y2 + y2z2 + z2x2) + k(xy3 + yz3 + zx3) > 2k(x3y + y3z + z3x) + (fc2 — k)xyz(x + y + z). (í). Cho k — 1 ta được X4 + y4 + z4 + xif + yz3 + zx3 > 2(x3y + y3z + z3x). (iiỴ Cho k — 2 ta được (x2 + y2 + z2)2 + (x2y2 + y2z2 + Z2X2) + (xy3 + yz3 + zx3) > 4(x3j/ + y3z + z3z) + 2xyz{x + y + z) (x2 + ý2 + z2 + xy + yz + zx)2 > 6(x3y + y3z + z3x) + 6xyz[x + y + z). Vậy nếu ó2 + b2 + c2 + ab + bc + ca = 6 thì a3b + b3c + c3a + abc(a + b + c) < 6. . Xét khai triển sau ((a - ò)2 + 2c(a - c))2 + ((b - c)2 + 2a(ò - a))2 + ((c - a)2 + 2b(c - ò))2 > 0 6(a4 + b4 + c4) + 4aỏc(a + b + c) + 2(a2b2 + b2c2 + c2a2) > 12(a36 + b3c + c3a) a4 + b4 + c4 + ~(ab + bc + ca)2 > 2(a3ò + b3c + c3a). ố Vậy hằng số k tốt nhất (lớn nhất) cho bất đẳng thức ìằ k — 1/3. □ Nhận xét. Chú ý rằng trong các bất đẳng thưc trên, luôn có đẳng thức khi các bicn a, b, c bằng nhau nhưng mỗi bất đẳng thức luôn có một trường hựp đẳng thức đặc biệt khác, chẳng hạn ở câu (ỉ) có đẳng thức khi a = 2 cos 20 + 1 « 2.88, b — 2 cos 40 « 1.532, c = -1. Bài toán 2.97 (Phạm Kim Hùng). Cho các số thực dương a,b,c,d thoả mãn a + b + c + d = i. Chứng minh rằng với mọi hằng số dương k,n>2 thì (k + a”)(fc + òn)(fc + cn)(fc + dn) > (fc + l)4. LỜI Giải. Đối với bài toán này, cần có một nhận xét quan trọng giúp cho việc giái bài toán trở nên đơn giản hơn, đó là bất đẳng thức sẽ đúng với mụi k,n> 2 khi và chỉ khi bất đẳng thức đúng với fc,n = 2. Thật vậy, với mọi n > 2 thì /fc + an\2 > ik + a2\n \kT1/ \ A; + 1 / Két quả này có thể chứng minh dùng bất đẳng thức AM - GM nên sẽ dành cho bạn đọc. Đây chỉ là một ước lượng căn bản giữa 2 dạng trung bình bậc 2 và bậc n. Giả sử bất đẳng thức đúng khi k — n = 2. Thế thì nk + a2 a,o,ctd n/2 í I (r + ữi)(r + ỏi)(r + Ci)(r + di) > [r + yaiỏiCidộ Vr, ai,Ỉ>1,C1,dì. > 0. Lấy r = k - 2 và ai = 2 4- a2, bi = 2 + b2, C1 = 2 + c2, di = 2 + d2, vì aỵbiCidi > 1 theo giả thiết ỏ trên nen ta có (k + a2)(k + b2)(k + c2)(fc + d2) >(k + l)4. Phần còn lại ta sẽ chứng minh khi k — n = 2. Ta phải chứng minh (2 + ữ2)(2 + ò2)(2 + c2)(2 + d2) > 81. Cách làm ngắn gụn và đơn giản nhất là sử dụng định lí dồn biến mạnh S.M.V. Dễ thấy rằng ncu a + b < 2 thì (2 + a2)(2 + b2) > (2+ ị0)- ) . Thật vậy, bất đẳng thức trên tương đương với 2(a2 + ò2)-(a + 6)2>^-^-a2&2 10 o 16 > (a + b)2 + 4aỏ. Hiển nhiên đúng vì a + b c > b > a. Vì c + a < 2 nên từ chứng minh trên, kết hợp với định lí S.M.V ta chỉ phải chứng minh bài toán khi a = b = c = X < d = 4 - 3x. Bất đẳng thức trở thành (2 + x2)3 (2 + (4 - 3x)2) >81. Đặt f(x) = 31n(2 + X2') + In (2 + (4 - 3x)2). Ta phải chứng minh f(x) > 4ln3. } = 6x _ 6(4 -3x) x 2 + X2 2 + (4 — 3x)2 f’(x) = 0z + |= 4-3a;+ 2 o (x - 1) I 4 - — ) — 0. v V x(4 - 3x) J Nếu X Ỷ 1 thì a?(4 - 3rr) = 2. Nhưng theo bất đẳng thức AM — GM thì 3rr(4 - 3x) x(4 — 3x) X = 1 và từ đó dễ dàng suy ra maxXỄ|o,i] Ị{x) — 41n2. Đây là điều phải chứng minh. □ Nhận xét. Ta có thể làm tổng quát hơn bài toán đã cho. Chú.ý rằng với mọi k > 1 và a + b < 2 ta đều có (k + a2)(Ẳ: + ỏ2) > (k + —. Nên với mọi k > 1 thì biểu thức (k + a2)(fc + b2)(k + c2)(fc + d2) sẽ nhận giá trị nhỏ nhất khi a = b — c = X < 1. Bằng tính toán đơn giản ta có kết quả sau (k + a2)(fc + b2)(k + c2)(fc + d2) > min [(k + l)4, (k + o2)3 (& + (4 - 3ữ)2) ), Trong đó a < 1 là nghiệm nhỏ của phương trình x(4 - 3x~) = k. Ta có thể tính trực tiếp giá trị của ữ với điều kiện k < 4/3 là 2 - ự4 - 3fc Q = 3 ’ Chẳng hạn ta có thổ rút ra các trường hợp riêng rất thú vị sau đây Nếu k = 1 thì / 1 \3 1 n4 (1 + a2)(l + b2)(l + c2)(l + d2) > 10 Ợ + = -93-. Nếu k = 5/4 thì => (5 + 4a2)(5 + 4&2)(5 + 4c2)(5 + 4d2) > 6480. Còn nếu fc = 4/3 thì => (4 + 3a2)(4 + 3b2)(4 + 3c2)(4 + 3d2) > min ( 74, 1 = 74 = 2401. Đây cũng là các bài toán rất khó. Các bạn có thể tự kiểm nghiệm các kết quả này. Bài toán 2.98 (Phạm Kim Hùng). Cho a, b, c là các số thực dương. Chứng minh bất đẳng thức b2\2 , / , c2\2 — + ò+ — c / \ a / LỜI Giải. Chuyển bất đẳng thức về dạng a2 + b2 + c2 + 12(ữ3 + b3 + c3) a + b + c 2ab2 2bc2 2ca2 b4 c2 a4 —- 1 1 1 ĩ. + Tộ Tã c a b c? ír Sử dụng các đẳng thức sau cyc sym cyc ' ' cyc sym cyc a(b — c)2 c 3(q3 + b3 + c3) _ y> = + c)(b - c)2 a + b + c a + b + c sym Bất đẳng thức đã cho tương đương với • (ò - c)2 fl + + (c - à)2 (1 + ỵ) + (a - b)2 (1 + \ c J \ a/ \ b/ 2a(b - c)2 , 2b(c-a)2 2c(a - b)2 . . 2 , ,2 , _2\ + —IT—4- __i_ + ——L. + 2(ữỏ + bc + ca) - 2(a2 + b + c ) c a b 12(a3 + b3 + c3) ./„2 1,2 „2\ > —1 - 4(a2 + b + cz) a + b + c cyc , b + c 2a 1 - 4- - — - + — ]DÒ~C)2 cyc b2 4a 2(a + b) c2 a+ b + c c -4 > 0. Đặt C = ÈĨ 4. 4a , 2(a + b) _ . a c2 a + b + c c c2 + 46 2(6 + c) a2 a + b + c a - 4, sc = g + + _ 4 b2 a + b + c b Xét 2 trường hợp (i). c> b> a. Dỗ thấy sb > 0. C 4- C, - °2 4- ờ2 4. 4(a + 4_ 2(ò + c) . 2(a + b) Sa + b — ~2 + 72 ■*" „ , k , 4 ~ 1 a2 c2 a + b + c a c - 8 > 0, Vì c2 . b2 26 „ 2c , 2a 2b n —n 4—X > — > 2, 1—— >4, — >2. a2 c2 a a c a Ọ ~ _ c2 a2 , 4(6 + c) .2(6 + c) 2(a + c) - 8 > 0, Vì c2 a2 2c a2 + b2 — b 2b 2a 2c „ > 2, — + > 4, — > 2. a ■ b a => sa(b - c)2 + sb(a - c)2 + sc(a - b)2 > (Sa + sb)(b - c)2 + (Sb + sc)(a - 6)2 > 0. (u). a > b > c. De thấy Sa > 1, Sc > -1 + 5^3- So + 2S1=^ + i + ^±±L + iS^ + ?-12>0, a2 cz a + b + c a c Vi 8b + 4a _ 2b 2a t 2c 2a , „ , t , „ — 4, —- + ~—> 4, — + — > 4. a+o+c a c a c Neu 2b> a + c thl 4c2 b2 Sa + 4Sb + Sc>^- + ^ + az c^ 16ò + 4a + 4c 8(0+ c) 2(a + b) —-— 1 — 1 — 21 a + b + c a c > + ÈỈ + 8(b + c) + 2(q + V _ 13 ~ a2 c2 a c 4c2 . 16c 2a „ r— „ >-0- + — + — - Í0 > 2V32 - 10 > 0. a2 a c Có 2 khả năng xảy ra (1). Nếu a + c a - c. Nếu Sb > 0 ta có đpcm. Nếu Sb < 0 ta vẫn có Sa(b - c)2 + sb(a - c)2 + sc(a - ò)2 > (Sa + 45b + Sc)(6 - c)2 > 0. (2). Nếu a + c > 2b, khi đó ta sẽ chứng minh Sc + 25'b > 0. C 4.9a _ _ 2c2 4- a2 -C 86 + 4c . 4(b + c) . 2(a + c) 19 Sc + 2Sb — g(c) — —77 + 72 + H - 1 - 12. az tra + o + c a b Vì g(c) là hàm tăng (c > 0) nên p(c) > min{p(o), g(2b - a)}. +, Nếu a> 2b thì 5(c) > 5(0) = £2 + 8b bằng cách cộng các bất đẳng thức sau a 96 _ í a + b 96 _ A a 46 + •——r > 5, I — 1 —7 > 6 ) , — 0 a + b \ b a + b lb +, Neu 2b > a thì _z X /r>L X 862 a2 46 4a => Sa(b - c)2 + Sb(a - c)2 + Sc(a - b)2 > (So + 2Sb)(b - c)2 + (Sc + 2Sb)(a - Ò)2 > 0. Bất đẳng thi'Ic được chứng minh hoàn toàn. Đẳng thức xảy ra khi và chỉ khi a = b = c. □ • Bài toán 2.99 (Gabriel Dospinesscu). Cho n > 2 là một số nguyên dương và (11,0/2, ...,an là các số thực. Chứng minh rằng với một tập con khác rỗng s bất kì của {.1,2, ...,n} bất đẳng thức sau đây luôn đúng ) 2 < (ữj + ... + flj) . Bổ đề. LỜI Giải. Ta sử dụng bổ đề sau )2 - ■ 52 (ai + ... + ctj)2. Chứng Minh. Đặt Sị = Xi + X2 + ... + Xi, khi đó ^2 a2i+l = S1 + S3 — §2 + S5 — S4 + ... + S2fc+1 ~ s2k- 0<i<k Vậy vế trái bất đẳng thức có thể khai triển thành 2fc+i S2is2j+1 + 2 S2ịS2j + 2 52 s2i+ls2j+1 + 2, i=l ij i<j ì<j Và vế phải bất đẳng thức có thể khai triển thành 2k+l - Sjý = (2k + 1) 52 sỉ i<j i=l i<j Do đó sau khi tách bớt các số hạng giống nhau ở 2 vế ta được bất đẳng thức tương đương là 2fc+l 2k S2iS2j + 4 s2i+lS2j+l- i=l i<j i<j Tuy nhiên bất đẳng thức trên hiển nhiên đúng, ta chỉ cần cộng theo vế vác bất đẳng thức sau đây s2i + s2j 2s2iS2ji s2i+l + s2j+l > 2s2i+lS2j+l- BỔ đề được chứng minh xong. Vào bài toán, ta phải xét với tập s c {1,2,..., n}. Hiển nhiên nếu s không có các phần tử rời nhau (S — {í, i + 1,j}) thì ta có điều phải chứng minh, vì trong các số hạng của VC phải có chứa cả VC trái. Nếu s gồm các đoạn rời nhau, giả sử s = + 11 •••172173173 + 1...,741 •■•! 72m+b 72m+l + 11 •••i72m+2}’ Khi đó ta đặt Ò1 = + djl+í + ... + Ọj2 ^2 = «72 + 1 + ••• + «73 ^2fc+l - «72m+l + aj2m+l+l + ••• + «J2m+2 k 5"^ «i — ^27+1 — VP- ifzS j=0 Ngoài ra theo bổ đề thì k \ 2 5>J+1 < 52 (6i + ...+0j)2< yp- j=l Ị (vì mỗi số (6j + ... + bj)2 đã xuất hiện ở trong biểu thức vế phải). Bất đẳng thức được chứng minh xong. □ Bài toán 2.100 (Phạm Kim Hùng). Chứng minh rằng với a,b,c là các số thực không âm tùy ý ta có bất đẳng thức a3 b3 c3 ĩ>abc (a + 6)3 + (ò + c)3 + (c + ữ)3 (a + b)(b+ c)(c + a) ~ LỜI Giải. Rõ ràng bất đẳng thức có thể chuyển về dạng tương đương là 1 1 1 5 (1 + x)3 + (1 + y)3 + (1 + z)3 + (1 + x)(1 + y)(l + z) ~ Với X = b/a,y = c/ò, z — a/c và xyz = 1. Đặt 2 2 2 m = 1 — ——,71 = 1 — ——,p — 1 — —— 1, m, n, p 6 [—1,11. 1 + x 1+ỉ/ 1 + z L J Ta có „ (1 + 77i)(l + 71)(1 + p) — (1 - m)(l - 71)(1 -p')=>m + n + p + mnp — 0. Bất đẳng thức trở thành (1 - 77i)3 + (1 - n)3 + (1 - p)3 + 5(1 - 771)(1 - 71)(1 - p) > 8 o 3(m2 + TI2 + p2) 4- 5(mn + np + pm) > 3(m + n + p) + m3 + n3 + p3 3(tti2 + TI2 + p2) + 5(77171 + np + pm) > m3 + n3.+ p3 - 3mnp. Chú ý rằng nếu 77171 + np + pm > 0 thì VT > 0, còn nếu 77171 + np + mp 0. Vậy chỉ cần chứng minh bất đẳng thức khi 771 + 71 + p > 0. Đặt t — m + n + pvầu = mn + np + pm, bất đẳng thức trở thành 3(t2 — 211) + 511 > t(t2 — 311) o t2(3- í) + u(3í- 1) > 0 (1) Theo bất đẳng thức AM - GM thì 7712 + 712 + p2 > 3ịmnpị2/3 > -3mnp = 3(a + b + c) => t2 - 211 > 3t => 211 < t(t - 3) (2) Bất đẳng thức sẽ được dễ dàng chứng minh nếu u > 0, ta chỉ cần xét khi 11 0, thay (2) vào (1) tã chỉ cần chứng minh 2í2(3 - t) + t(t - 3)(3t - 1) > 0« t(3- t)(l -t) > 0 t(3 - t)(l + mnp) > 0. Nhưng bất đẳng thức trên hiển nhiên đúng vì 771, n,p e [-1,1]- Đẳng thức xảy ra khi 771 - n = p = 0 hoặc 771 — 71 — l,p = -1, hay a — b — c hoặc b/a = c/b = +oo hoặc các hoán vị. Bất đẳng thức được chứng minh. □ Bàn về sáng tạo bất đẳng thức Thế giới toán học còn vô vàn những câu hỏi không có lời giải đáp, những giả thiết mà bề ngoài tưởng chừng đơn giản lại thách thức các nhà toán học hàng 'trăm năm. Hãy nhớ đén bài toán Fecma nổi tiếng : "Chứng minh phương trình xn + yn — zn không có nghiệm nguyên dương trong tập số tự nhiên khi n> 3 ", chỉ có lời giải sau hơn 300 năm, đã tốn không biết bao nhiêu thời gian và trí tuệ của hàng trăm nhà toán học lớn khắp thế giới. Chúng ta, với kiến thức phổ thông, không nên chỉ biết giải các bài toán do người khác đặt ra mà sẽ tốt hơn nếu có thể tự phát triển các vấn đề cũ hoặc đặt ra các bài toán mới trong tầm kiến thức của mình. Đây là một việc rất ý nghĩa, bổ ích và thú vị. Trở lại với bất đẳng thức, đây là một đối tượng căn bản và quen thuộc với hầu hét các bạn học sinh phổ thông. Bất đẳng thức xuất hiện trong các kì thi tuyển sinh đại học, các kì thi chọn học sinh giỏi quốc gia và quốc tế thường được rất nhiều người quan tâm. Tất nhiên, việc sáng tạo ra các bất đẳng thức cũng là một việc đáng quam tâm và nhiều ý nghĩa. Bất đẳng thức cũ và mới Do sự tồn tại từ rất lâu của bất đẳng thức mà hệ thống bất dẳng thức ngày càng phong phú, đa dạng. Tuy nhiên, không phải mọi bài toán đặt ra đều có ý nghĩa thực sự, ta chỉ quan tâm nhiều hơn đến các bất đẳng thức sẽ để lại cho những ý nghĩa nhất định. Chúng ta hãy xem xét một số bất đẳrig thức được tạo ra bằng các phương pháp cũ, mà thông thường là suy ra từ những kết quả biết trước. Sau đây là một vài ví dụ Ví dụ 2.2.1. Chứng minh rằng với a, b, c, d là các số thực dương tuỳ ý thì ta có bất dắng thức 2a2 3b2 2c2 d2 > 6ữ + 36 + 2c + d 2a + b + 6b + c + 6c + 3d + 6d + a ~ 12 LỜI Giải. Tuy rằng cách phát biểu của bài toán hơn lộn xộn và rắc rối, nhưng bản chất thì rất đơn giản. Bất đẳng thức trên được suy ra trực tiếp từ kết quả sau a2 b2 c2 d2 > a + b + c+ d a+b b+c c+đ d+a~ 2 Sau đó ta thay a, b, c, d bởi a, 6/2, c/3, d/6. □ Việc thay đổi các biến số bằng các hàm số đơn giản đã làm bài toán khó hơn vì đã che giấu đi bản chất thật của bài toán. Ví dụ 2.2.2. Chứng minh bất đẳng thức sau với a,b,c> 0 a4c4d4 + b4c4ds + c8 + d4 > a4bc5 + b3c6d4 + c2d3 + c7ad. LỜI Giải. Bài toán gốc của nó là a4 + b4 4- c4 + d4 > 0?b + Ò3C + c3d + d3a Sau đó ta thay ữ, b, c, d bởi a, bc, 1/c, c/d. □ Từ một bất đẳng thức tuỳ ý, ta có khá nhiều biến đổi để tạo ra một bất đẳng thức mới mà kĩ thuật đổi biến càng khó thì vấn đề càng khó được tìm ra. Tuy nhiên trong cả quyển sách, chúng ta sẽ không dùng cách thức này để sáng tạo hoặc mới hoá một bất đẳng thức cũ, tất cả các bài toán đều được đưa vè dạng chuẩn tắc nhất. Bằng phương pháp sử dụng đẳng thức, Dào Hải Long (học sinh khối chuyên toán Tổng hợp, H@v Olympic Toán quốc tế) có một nhóm các bất đẳng thức khá thú vị. Ví dụ 2.2.3. Chứng minh rằng với mọi x,y,z £ R phân biệt thì 2 9 -2 X y z > 2 (y — z)2 (z — x)2 (x — y)2 — LỜI Giải. Bất đẳng thức trên nhìn khá kì quặc vì lại có hiệu bình phương ở diíới mẫu. Lời giải của tác giả bài toán trên là như sau.

Các file đính kèm theo tài liệu này:

  • docxgiao_trinh_sang_tao_bat_dang_thuc.docx
  • pdfhoi_qui_su_dung_bien_gia_9784_43366.pdf